Helpppp me plzzzzzzxzz

Helpppp Me Plzzzzzzxzz

Answers

Answer 1
Can you please take a better picture so I can see it better

Related Questions

Choose the smallest number 3 1/8 or 10/3

Answers

Answer:

10/3

Step-by-step explanation:

31/8= 3.8

10/3= 3.3

According to the number line, what is the distance between points A and B?

0 6 units
7 units
O 12 units
O 14 units

Answers

Answer:

14 units

Step-by-step explanation:

A = - 2, B = 12

Therefore,

d(A, B) = 12 - (-2) = 12 + 2 = 14 units

Can someone help me with this math homework please!

Answers

Answer:

Step-by-step explanation:

In step one she has added 1x to each side of the statement

in step two she has taken five away from both sides of the equation therefore simplifying the entire equation

in step three she divided both sides by three to have the unknown value on one side of the equation

What’s the equation of the line that passes through the point (-4,4) and has a slope of 3/4

Answers

Answer:

y-y1=m(x-x1)

y-4=3/4(x+4)

y=3/4x+7

What is the scale of the y-axis in this coordinate graph?


A. 1 tick mark represents 1 unit
B. 1 tick mark represents 8 units
C. 1 tick mark represents 12 units
D. 1 tick mark represents 16 units

Answers

Answer:

Obviously B

Step-by-step explanation:

4m square - 8mn+3n ( resolve into factors)​

Answers

Step-by-step explanation:

your answer will be ( 2m-3n)(2m-n)

Explanation is in the attachment

hope it is helpful to you

hope it is helpful to you

Step-by-step explanation:

[tex]4 {m}^{2} - 8mn + 3 {n}^{2} \\ 4 {m}^{2} - (2 + 6)mn + 3 {n}^{2} \\ 4 {m}^{2} - 2mn + 6mn + 3 {n}^{2} \\ 2m(2m - n) - 3n(2m - n) \\( 2m - 3n) \: (2m - n)[/tex]

On the unit circle, which of the following angles has the terminal point
coordinates.
A. 45
B. 135
C. 225
D. 315

Answers

Answer: C. 225

Step-by-step explanation:

find the domain of f(x)=sec(2x)

Answers

Answer:

*Refer the image attached

Step-by-step explanation:

*Refer the image attached

Greg buys 60 garden plants at a cost price of $2.00 each to sell in his shop. He sells 25 of them at the profit of 75% and 18 of them at the profit of 35%. He sells the rest of the plants for 4/5 of the cost price calculate the profit or loss he makes from selling 60 plants stating if it is a profit or loss

Answers

Answer:

$43.30 profit

Step-by-step explanation:

Total cost of plant:

60*2 = 120

Greg makes total of:

25*(2 + 0.75*2) + 18*(2 + 0.35*2) + (60 - 25 - 18)*2*4/5 = 163.3

Since Greg mare than cost, he has a profit and the amount is:

163.3 - 120 = 43.3

A cinema is doing a promotion to celebrate their 50th anniversary for 1 week. They give

away a free drink to every 98th customer, a free bag of popcorn to every 112th customer and

a free cinema ticket to every 224th customer. Which lucky customer will be the first to

receive all 3 items?​

Answers

Answer:

1,568 customer

Step-by-step explanation:

Find the lowest common multiple of 98, 112, and 224

98 = 98, 196, 294, 392, 490, 588, 686, 784, 882, 980, 1078, 1176, 1274, 1372, 1470, 1568, 1666

112 = 112, 224, 336, 448, 560, 672, 784, 896, 1008, 1120, 1232, 1344, 1456, 1568, 1680, 1792, 1904

224 = 224, 448, 672, 896, 1120, 1344, 1568, 1792, 2016, 2240

The lowest common multiple of 98, 112, and 224 is 1568

Therefore, the 1,568th customer will be the first to receive all 3 iitem

2 1/4 x 3 1/5 brainliest

Answers

Answer:

36/5

Step-by-step explanation:

9/4×16/5

144/20

36/5

hope this is helpful

Answer:

[tex]7\frac{1}{5}[/tex]

Step-by-step explanation:

1. start by turning the fractions improper fractions:

[tex]2\frac{1}{4} =\frac{9}{4}[/tex]

[tex]3\frac{1}{5} =\frac{16}{5}[/tex]

2. then multiply them together:

[tex]\frac{9}{4}[/tex] x [tex]\frac{16}{5}[/tex] = [tex]\frac{144}{20}[/tex]

3. then simplify the fraction:

[tex]\frac{144}{20}[/tex][tex]=\frac{36}{5}[/tex]

4. turn it into a proper fraction:

[tex]\frac{36}{5} =7\frac{1}{5}[/tex]

0.2(x + 20) – 3 > –7 – 6.2x

Answers

Answer:

x > - 1.25

Step-by-step explanation:

0.2(x + 20) – 3 > –7 – 6.2x

0.2x + 4 - 3 > - 7 - 6.2x

0.2x + 1 > - 7 - 6.2x

Collect like terms

0.2x + 6.2x > -7 - 1

6.4x > -8

x > - 8/6.4

x > - 1.25

Note:

The > didn't change because you didn't divide by a negative value

Inequality signs changes when divided by a negative value

Answer:

Step-by-step explanation:

-1.25

Can someone help me with this math homework please!

Answers

Answer:

1: the number of years since 2008 2. t is greater than or equal to 0 3. negative values 4. continuous

Step-by-step explanation:

1. t usually represents time

2. t must be greater than 0 as you can not go backward in time

3. the range must be positive as you can not have negative bobcats

4. its continuous because its a quadratic equation

1. no. of bobcats since 2008

2. greater than equal to 0

3 negative values

4. discrete because no. of bobcats cannot be broken into fraction.

pls help me don't know what to do

Answers

Answer:

x=15

Step-by-step explanation:

The 60 degree angle and the (x+45) degree angle are both the same degree because they are vertical angles.

So to solve, just subtract 45 from 60

60-45=15

That's your answer!

Hope this helps!

PLISSSSSSSS HELPPPPP!!!!!!!!!!!!!!!!!!!!!
I will give brainliest............

Answers

I know the square root of sixteen is 4
1- 4
2- 1.5
3- 8
4- 1
5- 5
6- 9

Which operation will solve the following word problem? Jeff earns $14.00 per hour, Tom earns half as much as Jeff. How much does Tom earn per hour?


Multiplication


Subtraction


Addition


Division

Answers

Answer:

The correct option is (d).

Step-by-step explanation:

Given that,

Jeff earns $14.00 per hour.

Tom earns half as much as Jeff.

We need to find the amount earn by Tom per hour.

Tom's amount = Jeff's amount/2

So,

[tex]T=\dfrac{14}{2}\\\\T=\$7[/tex]

So, Tom earn $7 per hour. Hence, division operation is used. Jeff's amount is divided by 2.

Find ∠MPN

Help me please

Answers

Answer:

[tex]22^{\circ}[/tex]

Step-by-step explanation:

Line [tex]\overline{PM}[/tex] is a diameter of the circle because it passes through the circle's center O. Therefore, arc [tex]\widehat{PLM}[/tex] must be 180 degrees, as these are 360 degree in a circle.

We can then find the measure of arc [tex]\widehat{LM}[/tex]:

[tex]\widehat{LP}+\widehat{LM}=180^{\circ},\\92^{\circ}+\widehat{LM}=180^{\circ},\\\widehat{LM}=88^{\circ}[/tex]

Arc [tex]\widehat{LM}[/tex] is formed by angle [tex]\angle LPM[/tex]. Define an inscribed angle by an angle with a point on the circle creating an arc on the circumference of the circle. The measure of an inscribed angle is exactly half of the measure of the arc it forms.

Therefore, the measure of [tex]\angle LPM[/tex] must be:

[tex]m\angle LPM=\frac{88}{2}=44^{\circ}[/tex]

Similarly, the measure of [tex]\angle LNP[/tex] must be:

[tex]m\angle LNP=\frac{92}{2}=46^{\circ}[/tex]

Angles [tex]\angle LPM[/tex] and [tex]\angle MPN[/tex] form angle [tex]\angle LPN[/tex], which is one of the three angles in [tex]\triangle LPN[/tex]. Since the sum of the interior angles of a triangle add up to 180 degrees, we have:

[tex](\angle MPN+\angle LPM)+\angl+ PLN+\angle LNP=180^{\circ},\\\angle MPN+44+46+68=180,\\\angle MPN=180-44-46-68,\\\angle MPN=\boxed{22^{\circ}}[/tex]

The area of the rectangular sandbox at Dave's school is 117 square feet. The sandbox has a width of 9 feet as shown in the diagram. What is the perimeter of the sandbox?

Answers

Answer:

ay bru ima tell yu dhis rn is c

Step-by-step explanation:

lidentify the domain of the function shown in the graph
O A 15257
O B. 19334
O C. 221
O D. All real numbers

Answers

Answer:

B.

Step-by-step explanation:

the visible line is the defined function.

this line goes from x=1 to x=4, and has the functional results from y=1 to y=7.

the domain is the valid interval of the input variable (typically x), while the range is the valid inescapable of the result variable (typically y).

so, B is the right answer.

The center of the circle is at the point
, and its radius is
units. The equation of this circle in standard form is
.

Answers

Is there a picture that goes with this?

What is the quotient of (x^3 - 3x^2 + 3x - 2) ÷ (x^2 - x + 1)?
O x - 2
O x + 2
O x- 4
O x + 1

Answers

Answer:

x-2

The choose (1)

Step-by-step explanation:

(x³-3x²+3x-2)÷(x²-x+1)

(x-2)(x²-x+1) ÷ (x²-x+1)

Delete (x²-x+1)

so = (x-2)

Evaluate 2y when y = 6y

Answers

I believe the answer is 12y
im sure the answer is 12y

D=22/7×d-90 Solve the equation
Find D​
Fast!

Answers

Answer:

D=22-90+22d/7

Step-by-step explanation:

D=22/7×d-90

D=-90+22d/7

Answer is- d=-630/-22x+7

please I need answer right now ppleassssee

D. -1/4+-2/3
with explanation pleaseeee​

Answers

Answer:

1 - 1/3

Step-by-step explanation:

Since 1/2 * 2/3 = 1/3, it should be the case that 1/3 divided by 2/3 gives 1/2 and that 1/3 divided by 1/2 gives 2/3. To divide fractions, we multiply the numerator by the reciprocal of the denominator, where the reciprocal of a number just interchanges the numerator and denominator of the number.

If a bus travel for 120 minutes at a speed of 75 kilometers per hour how far has the bus traveled?

Answers

Answer:

150 km

Step-by-step explanation:

Put the minutes into hours 120min is 2 hours.

Distance = speed * time

Distance = 75 * 2

Distance = 150

Answer:

150 kilometers

Step-by-step explanation:

if the bus is going 75 kilometers an hour and they traveled for 120 minutes (exactly 2 hours) then you would just multiply 75 by 2 to get 150 kilometers total.

Find value of x.
A. 110
B. 47
C. 68
D. 112

Answers

Answer:

B

Step-by-step explanation:

The sum of the inner angles of a quadrilateral is 360 degrees

135 + 110 + 68 + x = 360

313 + x = 360

x = 47 degrees

Answer:

47

Step-by-step explanation:

whole thing is 360 degrees

68 + 110 + 135 = 313

360 - 313 = 47

x looks small too (if you had to guess in a multiple choice question)

There are 35 times as many students at Wow University as teachers. When all the students and
teachers are seated in the 8544 seat auditorium, 12 seats are empty. How many students attend
Wow University?

Answers

Given:

There are 35 times as many students at Wow University as teachers.

When all the students and teachers are seated in the 8544 seat auditorium, 12 seats are empty.

To find:

The total number of students.

Solution:

Let x be the number of teachers at Wow University. So, the number of student is :

[tex]35\times x=35x[/tex]

When all the students and teachers are seated in the 8544 seat auditorium, 12 seats are empty.

[tex]x+35x=8544-12[/tex]

[tex]36x=8532[/tex]

[tex]x=\dfrac{8532}{36}[/tex]

[tex]x=237[/tex]

The number of total students is:

[tex]35x=35(237)[/tex]

[tex]35x=8295[/tex]

Therefore, the total number of students is 8295.

A statistics professor asked students in a class their ages. On the basis of this information, the professor states that the average age of all the students in the university is 24 years. This is an example of

Answers

Answer:

propbability ???

Step-bp explanation:

Answer:

Step-by-step explanation:

This is an example of a statistical mean.

PLS HELP SOON WILL MARK BRAINLYEST

A railroad tunnel is shaped like a semi-ellipse, as shown below. A semiellipse is shown on the coordinate plane with vertices on the x axis and one point of intersection with the positive y axis. The height of the tunnel at the center is 35 ft, and the vertical clearance must be 21 ft at a point 8 ft from the center. Find an equation for the ellipse.

Answers

According to the question

b= 35 and (8,21) lies on the ellipse

After calculation we get a= 10

equation for the ellipse.

[tex] \frac{ {x}^{2} }{100} + \frac{ {y}^{2} }{1225} = 1[/tex]

Hello, Brainly community!

This question is for all of those Calculus people out there.

The volume of a swimming pool is changing with respect to time, such that the volume is given by W(t), where W(t) is measured in cubic centimeters and t is measured in seconds. A tangent line is shown for W(t) at t = 3 seconds. Determine the best estimate for the value of the instantaneous rate of change of W(t) when t = 3.
(I've narrowed down the answer choices to 2, and just really need to find the right way of thinking to find the answer)

(A) W(lim t) as t goes to 3.
(B) [W(3.1) - W(2.9)] / 0.2.

Thank you in advance!

Answers

Answer:

(B)  [tex]\displaystyle \frac{W(3.1) - W(2.9)}{0.2}[/tex]

General Formulas and Concepts:

Calculus

Limits

Derivatives

The definition of a derivative is the slope of the tangent line.

Derivative Notation

Instantaneous Rates

Tangent Line: [tex]\displaystyle f'(x) = \frac{f(b) - f(a)}{b - a}[/tex]

Step-by-step explanation:

Since we are trying to find a rate at which W(t) changes, we must find the derivative at t = 3.

We are given 2 close answer choices that would have the same numerical answer but different meanings:

(A)  [tex]\displaystyle \lim_{t \to 3} W(t)[/tex](B)  [tex]\displaystyle \frac{W(3.1) - W(2.9)}{0.2}[/tex]

If we look at answer choice (A), we see that our units would simply just be volume. It would not have the units of a rate of change. Yes, it may be the closest numerically correct answer, but it does not tell us the rate at which the volume would be changing and it is not a derivative.

If we look at answer choice (B), we see that our units would be cm³/s, and that is most certainly a rate of change. Answer choice (B) is also a derivative at t = 3, and a derivative tells us what rate something is changing.

∴ Answer choice (B) will give us the best estimate for the value of the instantaneous rate of change of W(t) when t = 3.

Topic: AP Calculus AB/BC (Calculus I/I + II)

Unit: Differentiation

Book: College Calculus 10e

Other Questions
Which should equal 105 to prove that f I g?abcd Note: Some writers omit the comma before and in a series. If you omit the final comma before and, do so consistently in your writing.EXERCISE 14 Using commas to separate independent clauses and items in a series Read the sentences. Add commas to separate independent clauses and items in a series.1 Jack his brother and I are going hiking and fishing this weekend.2 We had wanted to go to the beach but we couldn't get hotel accommodations.3 We decided to go to the mountains instead and enjoy the cooler weather.4 We'll probably hike for a few days and then we'll relax by fishing at the local lake.5 We'll take food sleeping bags a few changes of clothes but little else.6 Would you like to come along or are you busy this weekend?Follow-up Write two sentences illustrating Guideline 3 and two illustrating Guideline 4. Write about a recent vacation.Guideline Selecting a few households from New York City and observing whether or not they own stocks when it is known that 28% of all households in New York City own stocks. Is this experiment a binomial experiment? Explain why. provide three critical evaluations of your own contribution to environmental health and safety in your community Globalization have impacted all of the following EXCEPT.*1 pointOur eating habitsOur healthOur overall lifestyleour places of worshipAll of the following are examples of cultural diffusion of the Jamaican culture EXCEPT*1 pointWhites wearing braided hair and beadsWhite RastafariansJapanese Dancehall Queen and Reggae ArtistJamaicans eating ackee and saltfish Which one of the following statements concerning resistors in "parallel" is true? Question 7 options: The voltage across each resistor is the same. The current through each resistor is the same. The total current through the resistors is the sum of the current through each resistor. The power dissipated by each resistor is the same. What does the verse in Song of Solomon 5:16 mean His mouth is most sweet thank you very much How do you graph this helppp and explain Now actually compute 2 - 8 A balloon contains 0.118 mol of gas and has a volume of 2.58 L . If an additional 0.116 mol of gas is added to the balloon (at the same temperature and pressure), what will its final volume be? Can you also show the work so I can understand why is it that answer. thank you A literary device that compares two things using "like" or "as" is a calculate limits x>-infinity-2x^5-3x+1 On Mar 3, Lyons Company paid dividends of $1,000. Use your knowledge of what a correct journal entry should look like to identify what would be include Write a recursive rule for the sequence.x, x, 2x, 3x, 5x, 8x, ...I know that it adds its last term but I don't know the rule/formula to show that. Learning Task 4 Imagine you are the principal in an institution. As the school head, how are you going to address the social issues that circulate in your school? In your pad paper, copy the table. Write in the second column your suggested solution to the social issue indicated in the first column. Social Issues 1. Gender Bias /Discrimination 2 Overpopulation 3. Bullying 4. Fake News 5. Child Abuse Proposed Solution the angle of elevation of the top of the mast from a point 53m to its base on level ground is 61. find the height of the mast to the nearest meter Simplify 6/x^22x/x^2+3. How do I simplify rational expressions !!!PLEASE HELP!!!explain the error Imagine an anticline has been eroded to a flat surface. How would the rock age change as you walked across that flat surface